Search found 526 matches


you had 80 liters of water initially.

Observe that it is becoming 3/4 th of the solution in the second case.

=> 80 = 3/4 X OR 320/3 is the total solution in the second case.

you had 100 before, so the quantity added 100 - 320/3 = 20/3 :)

by harshavardhanc

Mon Jul 19, 2010 10:14 am
Forum: Problem Solving
Topic: mixture problem
Replies: 9
Views: 1938

What was the value of the sales of Anderson Books in 2000? (1) The sales of Anderson Books grew by $25,000 each year from 1990 to 2000. (2) The value of the sales of Anderson Books doubled between 1990 and 2000. Beyond just answering, can someone write out a detailed explanation if its not too much...

by harshavardhanc

Mon May 17, 2010 11:08 am
Forum: Data Sufficiency
Topic: Anderson Books
Replies: 6
Views: 1788

Rohan bought a bike for Rs.70000. This total amount was paid in installments. After paying the initial deposit of Rs 25000 by Rohan, the rest of the amount was paid by Arun, Rohan and his wife in the ratio of 4:4:1 every month for six months. After six months the bike met with an accident and had t...

by harshavardhanc

Mon May 17, 2010 4:49 am
Forum: Problem Solving
Topic: Ratios- installments on bike.
Replies: 6
Views: 1405

Well, in your method 3, 12 * 10 * 8 * 6 shows all possible arrangements of the four chosen cards, whereas we are looking for total combinations of those four cards, and that nCr = nPr/r!, therefore we need to divide 12 * 10 * 8 * 6 by 4! for the correct answer to the question. did you mean 12*10*8*...

by harshavardhanc

Mon May 17, 2010 4:09 am
Forum: Problem Solving
Topic: Probability
Replies: 4
Views: 1379

kstv wrote:Combining 1 and 2
r=s=t or rst = r^3
3 of pqrst are integers , has to be r,s and as they are =
pqr^3 = 4
pq= 4/r^3
r^3 can never be 4
so p cannot be 1/q
if we're getting a definite NO, won't the answer be C in that case.

by harshavardhanc

Mon May 17, 2010 12:28 am
Forum: Data Sufficiency
Topic: tricky DS question
Replies: 23
Views: 12415

http://s1.postimage.org/GJmlS.jpg The answer will be 2 PI. Solution : as PQ|| OR, <RPQ will be 35 degrees. Now the property to be used here: angle subtended by an arc at the center is twice the angle subtended by the arc on the circumference. therefore, minor arcs OP and QR will subtend 70 degrees ...

by harshavardhanc

Sun May 16, 2010 1:06 pm
Forum: Problem Solving
Topic: Circles
Replies: 3
Views: 1663

Any sight would be great. I have been trying to solve the problem using the group 1+group 2+ neither + both, but its not working. Can you solve using a variation of this: 24+11+1.5x+x=84? Thanks, Chris you were very close! the equation will become : 24 + x + 2/3 x = 84 , where x is the number of pe...

by harshavardhanc

Sun May 16, 2010 7:50 am
Forum: Problem Solving
Topic: Venn Diagram
Replies: 6
Views: 1532

We are told that these herbs are closely related, so it seems likely that they all produce histidine. Thus C seems to give us more of the same evidence. Assumption 1: "closely related " stretched too much. Humans and chimps are closely related. Do we put all our limbs on the ground while ...

by harshavardhanc

Sat May 15, 2010 12:36 pm
Forum: Critical Reasoning
Topic: Histadine
Replies: 24
Views: 5182

ern5231 wrote:2x = 3y-5 and y ≠ 3, then (x-2) / (y-3) =?

the answer to this is 3 / 2. How is this possible to get??
2x = 3y - 5

subtract 4 frm both sides

2x-4=3y-5-4

2x-4=3y-9

2(x-4) = 3 (y-3)

(x-4)/(y-3) = 3/2

by harshavardhanc

Sat May 15, 2010 7:14 am
Forum: Problem Solving
Topic: 318) How to get this one?
Replies: 2
Views: 948

sanju09 wrote: I slightly doubt, anybody!
me too!

grrrrr....made a mistake....jumped the gun..... :(

[spoiler]the answer will be 25%

1 in 4 parts is the crust so (1-x)/(4-x) = 1/5 :) is it correct now?[/spoiler]

by harshavardhanc

Sat May 15, 2010 5:30 am
Forum: Problem Solving
Topic: pumpkin pie
Replies: 5
Views: 1322

sanju09 wrote:The crust of a certain pumpkin pie is 25 percent of the pie. By what percent should the amount of crust be reduced in order to make it 20 percent of the pie?
(A) 5
(B) 10
(C) 15
(D) 20
(E) 25
25 - > 20

[spoiler]or a decrease of 5 over 25 = 20 % [/spoiler]

by harshavardhanc

Sat May 15, 2010 5:01 am
Forum: Problem Solving
Topic: pumpkin pie
Replies: 5
Views: 1322

Reasons why E is correct : "Yes, strengthens the argument. It implies that the plant has been producing histidine for a long time, to nullify the metallic affect, and hence as the plant reaches maturity, the plant's production of histidine has decreased. If histidine weren't the cause for null...

by harshavardhanc

Sat May 15, 2010 4:51 am
Forum: Critical Reasoning
Topic: Histadine
Replies: 24
Views: 5182

Hi Kevin, How do you know that the histidine doesn't stay contained within the plant, rendering the metals chemically inert? Domnu agree with Domnu. The stimulus gives us no clues about the place where reactions, similar to those in labs, occur in plants. If they're outside plants, then D may suppo...

by harshavardhanc

Sat May 15, 2010 4:42 am
Forum: Critical Reasoning
Topic: Histadine
Replies: 24
Views: 5182

Challenging the hypothesis that tuberculosis could only be transmitted when someone inhaled air that had been exhaled by an infected individual, an article published in Nature magazine in 1872 reported that when examined, monkeys that had been kept in confinement and had consequently never breathed...

by harshavardhanc

Sat May 15, 2010 4:22 am
Forum: Sentence Correction
Topic: Knewton SC challenge
Replies: 8
Views: 1962

Working alone, Maria can complete a task in 100 minutes. Shaniqua can complete the same task in two hours. They work together for 30 minutes when Liu, the new employee, joins and begins helping. They finish the task 20 minutes later. How long would it take Liu to complete the task alone? (A) 120 mi...

by harshavardhanc

Sat May 15, 2010 2:59 am
Forum: Problem Solving
Topic: alone, Maria can complete
Replies: 4
Views: 5943